5
$\begingroup$

I just bumped into the stochastic integral $$ \int_\rho^1 (W_t - W_{t-\rho}) \,dW_t $$ where $0 < \rho < 1$ is a constant and $W$ is a standard Wiener process. It would be nice if we have a closed-form representation of the cumulative distribution function, but for my work it suffices to show that there is some simple transform to make its law $\rho$-invariant, that is, to make its law not dependent on $\rho$.

Does it appear possible to any one? Thanks for any help in advance!


Additional Information: I guessed so because when $\rho$ is large enough, $\int_\rho^1 (W_t - W_{t-\rho})\,dW_t \approx W_\rho (W_1 - W_\rho)$. I tried to multiply it by $\frac{1}{\sqrt{\rho(1-\rho)}}$, but in some simple Monte Carlo experiments the distribution of the stochastic integral doesn't seem to be $\rho$-invariant.

$\endgroup$
2
  • $\begingroup$ What do you mean by transform? $\endgroup$
    – user69208
    Aug 13, 2018 at 8:22
  • 2
    $\begingroup$ @ZacharySelk If $I(\rho)$ denote the integral, I am wondering if $\frac{I(\rho)-f(\rho)}{g(\rho)}$ for some deterministic functions $f$ and $g$. $\endgroup$
    – Dormire
    Aug 13, 2018 at 9:26

2 Answers 2

1
$\begingroup$

(This is not a complete answer, rather an attempt to describe the distribution of the integral.)


The integral $\int_\rho^t W(t) dW(t)$ is simply $\tfrac{1}{2} ((W(1))^2 - 1) - \tfrac{1}{2} ((W(\rho))^2 - \rho)$. Let us handle the other term: $$I_\rho = \int_\rho^1 W(t-\rho) dW(t).$$ Let us approximate $W(t-\rho)$ by an elementary process $X_n(t)$, which is equal to $0$ for $t \in [0, \rho)$, and to $W_{k \rho/n - \rho}$ for $t \in [\tfrac{k}{n} \rho, \tfrac{k+1}{n} \rho)$, $k = n, n + 1, n + 2, \ldots$ Clearly, $$ I = \lim_{n \to \infty} \int_\rho^{K(n) \rho / n} X_n(t) dW(t) ,$$ where $K(n) = \lfloor n/\rho \rfloor$. Furthermore, $$ I_n := \int_\rho^{K(n) \rho / n} X_n(t) dW(t) = \sum_{k = n}^{K(n)} W(\tfrac{k}{n} \rho - \rho) (W(\tfrac{k + 1}{n} \rho) - W(\tfrac{k}{n} \rho)) $$ By scaling, $$ I_n \stackrel{D}{=} \frac{\rho}{n} \sum_{k = n}^{K(n)-1} W(k - n) (W(k + 1) - W(k)) . $$ Write $Y_k = W(k+1) - W_k$, so that $Y_k$ is an i.i.d. sequence of standard Gaussian random variables. Then $$ I_n \stackrel{D}{=} \frac{\rho}{n} \sum_{k = n}^{K(n)-1} \sum_{j = 0}^{k - n - 1} Y_j Y_k. $$ It follows that $I_n$ is a certain quadratic form, evaluated on a standard Gaussian vector. If $\lambda_{n,j}$ denote the eigenvalues of the corresponding (symmetric) matrix $$ A_n = [\tfrac{\rho}{2 n} \mathbb{1}_{|j-k| \geqslant n}]_{j,k = 1}^{K(n)}, $$ then $$ I_n \stackrel{D}{=} \sum_{j = 1}^{K(n)} \lambda_{n,j} Z_j^2, $$ with $Z_j$ another i.i.d. sequence of standard Gaussian random variables.


Numerically, the eigenvalues $\lambda_{n,j}$ (arranged in the decreasing order of their absolute values) appear to converge to a certain limit $\lambda_j$, which essentially depends on $\rho$. This would mean that $$ I \stackrel{D}{=} c + \sum_{j = 1}^{\infty} \lambda_j (Z_j^2 - 1), $$ with $\lambda_j$ (and possibly $c$) essentially dependent on $\rho$. This, in turn, implies that there is no simple transform of $I$ that would make its distribution invariant with respect to $\rho$.


The numbers $\lambda_j$ are the eigenvalues of the integral operator with kernel $K(s, t) = \tfrac{\rho}{2} \mathbb{1}_{|s - t| > \rho}$ (acting on $L^2([0,1])$). However, I do not know if this helps in establishing a closed-form expression for $\lambda_j$.

$\endgroup$
-1
$\begingroup$

From $\int_0^1W_t\,dW_t={1\over 2}(W_t^2-t)$. One can deduce with the change of variable $s=t-\rho$, that your integral is equal to: $$ {1\over 2}(W_1^2-W_\rho^2-W_{1-\rho}^2) $$

$\endgroup$

Your Answer

By clicking “Post Your Answer”, you agree to our terms of service and acknowledge you have read our privacy policy.

Not the answer you're looking for? Browse other questions tagged or ask your own question.